Question

6. Consider two spin particles. Let nbe a unit vector joining the particles and consider the operator S12-3(? i)(o, . n ) _ (? ?2 ) . Show that for the singlet state its eigenvalue is 0. Show that in the triplet state its eigenvalues are -4 or 2. Choose along the z axis. (The Pauli principle is not considered.)

0 0
Add a comment Improve this question Transcribed image text
Answer #1

Add a comment
Know the answer?
Add Answer to:
6. Consider two spin particles. Let nbe a unit vector joining the particles and consider the...
Your Answer:

Post as a guest

Your Name:

What's your source?

Earn Coins

Coins can be redeemed for fabulous gifts.

Not the answer you're looking for? Ask your own homework help question. Our experts will answer your question WITHIN MINUTES for Free.
Similar Homework Help Questions
  • System A consists of two spin-1/2 particles, and has a four-dimensional Hilbert space. 1. Write down...

    System A consists of two spin-1/2 particles, and has a four-dimensional Hilbert space. 1. Write down a basis for the Hilbert space of two spin-1/2 particles. 2. Calculate the matrix of the angular momentum operator, Sfot = (ŜA, ŠA, ŜA) for system A, in the basis of question 4A.1, and express them in this basis. 3. Calculate the square of the total angular momentum of system A , Spotl?, and express this operator in the basis of question 4A.1. 4....

  • 2. Spin-1/2 system: (20 points) The Pauli matrices are, 0 -1 from which we can define...

    2. Spin-1/2 system: (20 points) The Pauli matrices are, 0 -1 from which we can define the spin matrices, s.-슬&z, Šv = , S.-출.. We'll use the eigenkets of S that, for the spin half system, they can be represented by the spinors, a) Show, by matrix multiplication that |+) and |-) are eigenstates of the S operator and determine the eigenvalues. Show that they are not eigenstates of S and Sy b) Show that the matrix squares s ,...

  • 1. In this problem, we are going to look at a three-level system. A spin-1 particld is placed in ...

    1. In this problem, we are going to look at a three-level system. A spin-1 particld is placed in a constant magnetic field along the a-direction with strength B,. The spin-1 particle İs initialized in a z-eigenstate with positive eigenvalue h, ie, the i 1,m 1) state. What is the probability to find the negative eigenvalue the spin along the z axis as a function of time? Assume that the spin-1 particle has inagnetic moment 2 × μιι, i.e. that...

  • A beam of spin particles is sent through a series of three Stern-Gerlach filtering devices. Initially the beam of parti...

    A beam of spin particles is sent through a series of three Stern-Gerlach filtering devices. Initially the beam of particles has equal numbers of particles with S, = h/2 and S.--h/2. The first filter (F+ transmits particles with S. h/2 and filters out particles with S, =-h/2. The second filter (Fr.) transmits particles with Sn-h/2 and filters out particles with S-h/2. A last filter (F- transmits particles with S--h/2 and filters out particles with S,-h/2 (i) Consider the state 2)12...

  • Consider one dimensional lattice of N particles having a spin of 1 /2 with an associated magnetic...

    Consider one dimensional lattice of N particles having a spin of 1 /2 with an associated magnetic moment μ The spins are kept in a magnetic field with magnetic induction B along the z direction. The spin can point either up, t, or down, , relative to the z axis. The energy of particle with spin down is e B and that of particle with spin up is ε--B. We assume that the system is isolated from. its environment so...

  • Consider the state of a spin-1/2 particle 14) = v1o (31+z) + i] – z)) where...

    Consider the state of a spin-1/2 particle 14) = v1o (31+z) + i] – z)) where | z) are the eigenstates of the operator of the spin z-component $z. 1. Show that [V) is properly normalized, i.e. (W14) = 1. 2. Calculate the probability that a measurement of $x = 6x yields 3. Calculate the expectation value (Šx) for the state 14) and its dispersion ASx = V(@z) – ($()2. 4. Assume that the spin is placed in the magnetic...

  • qm 2019.3 3. The Hamiltonian corresponding to the magnetic interaction of a spin 1/2 particle with...

    qm 2019.3 3. The Hamiltonian corresponding to the magnetic interaction of a spin 1/2 particle with charge e and mass m in a magnetic field B is À eB B. Ŝ, m where Ŝ are the spin angular momentum operators. You should make use of expres- sions for the spin operators that are given at the end of the question. (i) Write down the energy eigenvalue equation for this particle in a field directed along the y axis, i.e. B...

  • 3. (6 points) Measurements on a two-particle state Consider the state for a system of two...

    3. (6 points) Measurements on a two-particle state Consider the state for a system of two spin-1/2 particles, (2]+).I+)2 +1-)[+)2-1-)1-)2). (a) Show that this state is normalized. (b) What is the probability of measuring S: (the z-component of spin for particle 1) to be +h/2? After this measurement is made with this result, what is the state of the system? If we make a measurement in this new state, what is now the probability of measuring S3 = +h/2? (e)...

  • Answer all of the following questions and show works: (must show works) Problem 1 Consider a...

    Answer all of the following questions and show works: (must show works) Problem 1 Consider a particle in the state [4) = C( 31+z) – iV7|-z)). Note:77 ~ 2.646. a. What is the value of C? Assume C is a real, positive number. b. 1) Counter A A stream of 5000 particles, all in state 14) (from part a) enter a Stern-Gerlach analyzer as shown on the right. Counter B What will counter A read, on average? Counter C c....

  • 3. A particle with mass m and charge q moves in a uniform magnetic filed of...

    3. A particle with mass m and charge q moves in a uniform magnetic filed of magnitude B that is oriented along the z axis. (a) Neglecting the effects of spin and using the so-called Landau gauge with the vector po- tential given by A = (-By,0,0), show that the Hamiltonian may be written as À = 2m 2 ++øp +29BD2y +(, 2] (1) с (b) Because Pa and Êz commute with Ĥ, the time-independent Schrödinger equation for (x, y,...

ADVERTISEMENT
Free Homework Help App
Download From Google Play
Scan Your Homework
to Get Instant Free Answers
Need Online Homework Help?
Ask a Question
Get Answers For Free
Most questions answered within 3 hours.
ADVERTISEMENT
ADVERTISEMENT
ADVERTISEMENT